Proving limit of $a^nto0$ for $|a|<1$ without use of logarithms

The name of the pictureThe name of the pictureThe name of the pictureClash Royale CLAN TAG#URR8PPP











up vote
2
down vote

favorite












Prove that $a^nto0$ as $nto∞$ for $|a|<1$ without use of logarithms by using properties of the sequence $u_n=|a|^n.$



I've noticed that I should use the subsequence $u_2n$, and the fact that $u_2n=u_n^2$. However, I don't know where to go from here. I'm not familiar with these types of proofs so a hint/solution would be greatly appreciated.



Thank you!







share|cite|improve this question























    up vote
    2
    down vote

    favorite












    Prove that $a^nto0$ as $nto∞$ for $|a|<1$ without use of logarithms by using properties of the sequence $u_n=|a|^n.$



    I've noticed that I should use the subsequence $u_2n$, and the fact that $u_2n=u_n^2$. However, I don't know where to go from here. I'm not familiar with these types of proofs so a hint/solution would be greatly appreciated.



    Thank you!







    share|cite|improve this question





















      up vote
      2
      down vote

      favorite









      up vote
      2
      down vote

      favorite











      Prove that $a^nto0$ as $nto∞$ for $|a|<1$ without use of logarithms by using properties of the sequence $u_n=|a|^n.$



      I've noticed that I should use the subsequence $u_2n$, and the fact that $u_2n=u_n^2$. However, I don't know where to go from here. I'm not familiar with these types of proofs so a hint/solution would be greatly appreciated.



      Thank you!







      share|cite|improve this question











      Prove that $a^nto0$ as $nto∞$ for $|a|<1$ without use of logarithms by using properties of the sequence $u_n=|a|^n.$



      I've noticed that I should use the subsequence $u_2n$, and the fact that $u_2n=u_n^2$. However, I don't know where to go from here. I'm not familiar with these types of proofs so a hint/solution would be greatly appreciated.



      Thank you!









      share|cite|improve this question










      share|cite|improve this question




      share|cite|improve this question









      asked Aug 2 at 9:35









      RJM

      133




      133




















          7 Answers
          7






          active

          oldest

          votes

















          up vote
          2
          down vote



          accepted










          Just for simplicity consider the case $0<a<1.$ Now consider the sequence $u_n=a^n.$ Show that



          1. $u_n$ is decreasing;

          2. $u_n$ is bounded from below.

          Thus we have that $u_n$ is convergent. Let's write $L=lim_n u_n.$ Since $u_2n=u_n^2$ we have that



          $$lim_n u_2n=L=L^2=lim_n u_n^2.$$ So $L=L^2$ from where $L=0$ or $L=1.$ The case $L= 1$ is not possible since $u_n<1$ and it is decreasing.






          share|cite|improve this answer























          • Sorry if I'm missing something obvious, but how is it that the limit of u_n is equal to the limit of u_n^2?
            – RJM
            Aug 2 at 9:53










          • I had a typo. I meant $lim_n u_2n=L=L^2=lim_n u_n^2.$ (If $u_n$ is convergent to $L$ then $L=lim_n u_n=lim_n u_2n=lim_n u_2n+7$ because $u_2n, u_2n+7$ are subsequences of a convergent sequence and thus convergent to the same limit.)
            – mfl
            Aug 2 at 9:56


















          up vote
          2
          down vote













          If $|a|<1$, then $1/|a| >1$, hence there is $x>0$ such that $frac1a=1+x$. With Bernoulli we get for $ n in mathbb N$:



          $frac1a^n=(1+x)^n ge 1+nx > nx$.



          Hence



          $|a^n| <frac1nx$.



          Conclusion: $a^n to 0$.






          share|cite|improve this answer




























            up vote
            1
            down vote













            Assume $0le a<1$ and set $b=frac1a >1$ then by Bernoulli inequality



            $$b^n=(1-(b-1))^nge 1-n(b-1) to infty$$



            and then



            $$a^n=frac1b^nto 0$$



            More in general for $|a|<1$ we have that



            $$-|a|^nle a^nle |a|^n$$



            and then by squeeze theorem we conclude that



            $$a^n to 0$$






            share|cite|improve this answer






























              up vote
              1
              down vote













              For fun:



              1) $a_n= |a|^n$ is a strictly decreasing sequence for $ 0 <|a| lt 1$.



              2) $a_n$ is bounded below by $0$.



              3) Hence convergent.



              4)Recursively defined by:



              $a_n+1 =|a|a_n$, with $a_1=|a|$.



              5) $lim_n rightarrow infty a_n+1 = lim_n rightarrow infty(|a|a_n)$.



              With $lim_n rightarrow infty a_n+1 =$



              $ lim_n rightarrow infty a_n =:L ge 0$,



              we have:



              6) $L=|a|L$ with $0 < |a| lt 1 $.



              Hence?






              share|cite|improve this answer






























                up vote
                0
                down vote













                Note that $$ |a^n+1| = |a||a^n|$$



                The sequence $a^n$ is decreasing and bounded below so it converges to $L$



                Taking limit of $$ |a^n+1| = |a||a^n|$$ we get $|a|L=L$ which implies $L=0$






                share|cite|improve this answer




























                  up vote
                  0
                  down vote













                  Why not just (assuming as mfl did that $0<a<1$) use definitions:
                  $a^n$ is decreasing and bounded below, hence convergent, as in mfl's post, and the limit $L = inf a^n$ is pretty clearly nonnegative. Assume by way of contradiction that $L>0$. Then there exists $N>0$ such that $nge N$ implies $a^n < L(1/a)$. Then $a^N+1 < L$, contradiction.






                  share|cite|improve this answer




























                    up vote
                    0
                    down vote













                    By ratio test



                    $$frac^n+1=|a|<1 implies |a|^n to 0$$



                    then since



                    $$-|a|^nle a^nle |a|^n$$



                    by squeeze theorem we conclude that



                    $$a^n to 0$$






                    share|cite|improve this answer





















                      Your Answer




                      StackExchange.ifUsing("editor", function ()
                      return StackExchange.using("mathjaxEditing", function ()
                      StackExchange.MarkdownEditor.creationCallbacks.add(function (editor, postfix)
                      StackExchange.mathjaxEditing.prepareWmdForMathJax(editor, postfix, [["$", "$"], ["\\(","\\)"]]);
                      );
                      );
                      , "mathjax-editing");

                      StackExchange.ready(function()
                      var channelOptions =
                      tags: "".split(" "),
                      id: "69"
                      ;
                      initTagRenderer("".split(" "), "".split(" "), channelOptions);

                      StackExchange.using("externalEditor", function()
                      // Have to fire editor after snippets, if snippets enabled
                      if (StackExchange.settings.snippets.snippetsEnabled)
                      StackExchange.using("snippets", function()
                      createEditor();
                      );

                      else
                      createEditor();

                      );

                      function createEditor()
                      StackExchange.prepareEditor(
                      heartbeatType: 'answer',
                      convertImagesToLinks: true,
                      noModals: false,
                      showLowRepImageUploadWarning: true,
                      reputationToPostImages: 10,
                      bindNavPrevention: true,
                      postfix: "",
                      noCode: true, onDemand: true,
                      discardSelector: ".discard-answer"
                      ,immediatelyShowMarkdownHelp:true
                      );



                      );








                       

                      draft saved


                      draft discarded


















                      StackExchange.ready(
                      function ()
                      StackExchange.openid.initPostLogin('.new-post-login', 'https%3a%2f%2fmath.stackexchange.com%2fquestions%2f2869873%2fproving-limit-of-an-to0-for-a1-without-use-of-logarithms%23new-answer', 'question_page');

                      );

                      Post as a guest






























                      7 Answers
                      7






                      active

                      oldest

                      votes








                      7 Answers
                      7






                      active

                      oldest

                      votes









                      active

                      oldest

                      votes






                      active

                      oldest

                      votes








                      up vote
                      2
                      down vote



                      accepted










                      Just for simplicity consider the case $0<a<1.$ Now consider the sequence $u_n=a^n.$ Show that



                      1. $u_n$ is decreasing;

                      2. $u_n$ is bounded from below.

                      Thus we have that $u_n$ is convergent. Let's write $L=lim_n u_n.$ Since $u_2n=u_n^2$ we have that



                      $$lim_n u_2n=L=L^2=lim_n u_n^2.$$ So $L=L^2$ from where $L=0$ or $L=1.$ The case $L= 1$ is not possible since $u_n<1$ and it is decreasing.






                      share|cite|improve this answer























                      • Sorry if I'm missing something obvious, but how is it that the limit of u_n is equal to the limit of u_n^2?
                        – RJM
                        Aug 2 at 9:53










                      • I had a typo. I meant $lim_n u_2n=L=L^2=lim_n u_n^2.$ (If $u_n$ is convergent to $L$ then $L=lim_n u_n=lim_n u_2n=lim_n u_2n+7$ because $u_2n, u_2n+7$ are subsequences of a convergent sequence and thus convergent to the same limit.)
                        – mfl
                        Aug 2 at 9:56















                      up vote
                      2
                      down vote



                      accepted










                      Just for simplicity consider the case $0<a<1.$ Now consider the sequence $u_n=a^n.$ Show that



                      1. $u_n$ is decreasing;

                      2. $u_n$ is bounded from below.

                      Thus we have that $u_n$ is convergent. Let's write $L=lim_n u_n.$ Since $u_2n=u_n^2$ we have that



                      $$lim_n u_2n=L=L^2=lim_n u_n^2.$$ So $L=L^2$ from where $L=0$ or $L=1.$ The case $L= 1$ is not possible since $u_n<1$ and it is decreasing.






                      share|cite|improve this answer























                      • Sorry if I'm missing something obvious, but how is it that the limit of u_n is equal to the limit of u_n^2?
                        – RJM
                        Aug 2 at 9:53










                      • I had a typo. I meant $lim_n u_2n=L=L^2=lim_n u_n^2.$ (If $u_n$ is convergent to $L$ then $L=lim_n u_n=lim_n u_2n=lim_n u_2n+7$ because $u_2n, u_2n+7$ are subsequences of a convergent sequence and thus convergent to the same limit.)
                        – mfl
                        Aug 2 at 9:56













                      up vote
                      2
                      down vote



                      accepted







                      up vote
                      2
                      down vote



                      accepted






                      Just for simplicity consider the case $0<a<1.$ Now consider the sequence $u_n=a^n.$ Show that



                      1. $u_n$ is decreasing;

                      2. $u_n$ is bounded from below.

                      Thus we have that $u_n$ is convergent. Let's write $L=lim_n u_n.$ Since $u_2n=u_n^2$ we have that



                      $$lim_n u_2n=L=L^2=lim_n u_n^2.$$ So $L=L^2$ from where $L=0$ or $L=1.$ The case $L= 1$ is not possible since $u_n<1$ and it is decreasing.






                      share|cite|improve this answer















                      Just for simplicity consider the case $0<a<1.$ Now consider the sequence $u_n=a^n.$ Show that



                      1. $u_n$ is decreasing;

                      2. $u_n$ is bounded from below.

                      Thus we have that $u_n$ is convergent. Let's write $L=lim_n u_n.$ Since $u_2n=u_n^2$ we have that



                      $$lim_n u_2n=L=L^2=lim_n u_n^2.$$ So $L=L^2$ from where $L=0$ or $L=1.$ The case $L= 1$ is not possible since $u_n<1$ and it is decreasing.







                      share|cite|improve this answer















                      share|cite|improve this answer



                      share|cite|improve this answer








                      edited Aug 2 at 9:56


























                      answered Aug 2 at 9:43









                      mfl

                      22.9k11837




                      22.9k11837











                      • Sorry if I'm missing something obvious, but how is it that the limit of u_n is equal to the limit of u_n^2?
                        – RJM
                        Aug 2 at 9:53










                      • I had a typo. I meant $lim_n u_2n=L=L^2=lim_n u_n^2.$ (If $u_n$ is convergent to $L$ then $L=lim_n u_n=lim_n u_2n=lim_n u_2n+7$ because $u_2n, u_2n+7$ are subsequences of a convergent sequence and thus convergent to the same limit.)
                        – mfl
                        Aug 2 at 9:56

















                      • Sorry if I'm missing something obvious, but how is it that the limit of u_n is equal to the limit of u_n^2?
                        – RJM
                        Aug 2 at 9:53










                      • I had a typo. I meant $lim_n u_2n=L=L^2=lim_n u_n^2.$ (If $u_n$ is convergent to $L$ then $L=lim_n u_n=lim_n u_2n=lim_n u_2n+7$ because $u_2n, u_2n+7$ are subsequences of a convergent sequence and thus convergent to the same limit.)
                        – mfl
                        Aug 2 at 9:56
















                      Sorry if I'm missing something obvious, but how is it that the limit of u_n is equal to the limit of u_n^2?
                      – RJM
                      Aug 2 at 9:53




                      Sorry if I'm missing something obvious, but how is it that the limit of u_n is equal to the limit of u_n^2?
                      – RJM
                      Aug 2 at 9:53












                      I had a typo. I meant $lim_n u_2n=L=L^2=lim_n u_n^2.$ (If $u_n$ is convergent to $L$ then $L=lim_n u_n=lim_n u_2n=lim_n u_2n+7$ because $u_2n, u_2n+7$ are subsequences of a convergent sequence and thus convergent to the same limit.)
                      – mfl
                      Aug 2 at 9:56





                      I had a typo. I meant $lim_n u_2n=L=L^2=lim_n u_n^2.$ (If $u_n$ is convergent to $L$ then $L=lim_n u_n=lim_n u_2n=lim_n u_2n+7$ because $u_2n, u_2n+7$ are subsequences of a convergent sequence and thus convergent to the same limit.)
                      – mfl
                      Aug 2 at 9:56











                      up vote
                      2
                      down vote













                      If $|a|<1$, then $1/|a| >1$, hence there is $x>0$ such that $frac1a=1+x$. With Bernoulli we get for $ n in mathbb N$:



                      $frac1a^n=(1+x)^n ge 1+nx > nx$.



                      Hence



                      $|a^n| <frac1nx$.



                      Conclusion: $a^n to 0$.






                      share|cite|improve this answer

























                        up vote
                        2
                        down vote













                        If $|a|<1$, then $1/|a| >1$, hence there is $x>0$ such that $frac1a=1+x$. With Bernoulli we get for $ n in mathbb N$:



                        $frac1a^n=(1+x)^n ge 1+nx > nx$.



                        Hence



                        $|a^n| <frac1nx$.



                        Conclusion: $a^n to 0$.






                        share|cite|improve this answer























                          up vote
                          2
                          down vote










                          up vote
                          2
                          down vote









                          If $|a|<1$, then $1/|a| >1$, hence there is $x>0$ such that $frac1a=1+x$. With Bernoulli we get for $ n in mathbb N$:



                          $frac1a^n=(1+x)^n ge 1+nx > nx$.



                          Hence



                          $|a^n| <frac1nx$.



                          Conclusion: $a^n to 0$.






                          share|cite|improve this answer













                          If $|a|<1$, then $1/|a| >1$, hence there is $x>0$ such that $frac1a=1+x$. With Bernoulli we get for $ n in mathbb N$:



                          $frac1a^n=(1+x)^n ge 1+nx > nx$.



                          Hence



                          $|a^n| <frac1nx$.



                          Conclusion: $a^n to 0$.







                          share|cite|improve this answer













                          share|cite|improve this answer



                          share|cite|improve this answer











                          answered Aug 2 at 9:53









                          Fred

                          37k1237




                          37k1237




















                              up vote
                              1
                              down vote













                              Assume $0le a<1$ and set $b=frac1a >1$ then by Bernoulli inequality



                              $$b^n=(1-(b-1))^nge 1-n(b-1) to infty$$



                              and then



                              $$a^n=frac1b^nto 0$$



                              More in general for $|a|<1$ we have that



                              $$-|a|^nle a^nle |a|^n$$



                              and then by squeeze theorem we conclude that



                              $$a^n to 0$$






                              share|cite|improve this answer



























                                up vote
                                1
                                down vote













                                Assume $0le a<1$ and set $b=frac1a >1$ then by Bernoulli inequality



                                $$b^n=(1-(b-1))^nge 1-n(b-1) to infty$$



                                and then



                                $$a^n=frac1b^nto 0$$



                                More in general for $|a|<1$ we have that



                                $$-|a|^nle a^nle |a|^n$$



                                and then by squeeze theorem we conclude that



                                $$a^n to 0$$






                                share|cite|improve this answer

























                                  up vote
                                  1
                                  down vote










                                  up vote
                                  1
                                  down vote









                                  Assume $0le a<1$ and set $b=frac1a >1$ then by Bernoulli inequality



                                  $$b^n=(1-(b-1))^nge 1-n(b-1) to infty$$



                                  and then



                                  $$a^n=frac1b^nto 0$$



                                  More in general for $|a|<1$ we have that



                                  $$-|a|^nle a^nle |a|^n$$



                                  and then by squeeze theorem we conclude that



                                  $$a^n to 0$$






                                  share|cite|improve this answer















                                  Assume $0le a<1$ and set $b=frac1a >1$ then by Bernoulli inequality



                                  $$b^n=(1-(b-1))^nge 1-n(b-1) to infty$$



                                  and then



                                  $$a^n=frac1b^nto 0$$



                                  More in general for $|a|<1$ we have that



                                  $$-|a|^nle a^nle |a|^n$$



                                  and then by squeeze theorem we conclude that



                                  $$a^n to 0$$







                                  share|cite|improve this answer















                                  share|cite|improve this answer



                                  share|cite|improve this answer








                                  edited Aug 2 at 10:26


























                                  answered Aug 2 at 9:53









                                  gimusi

                                  63.8k73480




                                  63.8k73480




















                                      up vote
                                      1
                                      down vote













                                      For fun:



                                      1) $a_n= |a|^n$ is a strictly decreasing sequence for $ 0 <|a| lt 1$.



                                      2) $a_n$ is bounded below by $0$.



                                      3) Hence convergent.



                                      4)Recursively defined by:



                                      $a_n+1 =|a|a_n$, with $a_1=|a|$.



                                      5) $lim_n rightarrow infty a_n+1 = lim_n rightarrow infty(|a|a_n)$.



                                      With $lim_n rightarrow infty a_n+1 =$



                                      $ lim_n rightarrow infty a_n =:L ge 0$,



                                      we have:



                                      6) $L=|a|L$ with $0 < |a| lt 1 $.



                                      Hence?






                                      share|cite|improve this answer



























                                        up vote
                                        1
                                        down vote













                                        For fun:



                                        1) $a_n= |a|^n$ is a strictly decreasing sequence for $ 0 <|a| lt 1$.



                                        2) $a_n$ is bounded below by $0$.



                                        3) Hence convergent.



                                        4)Recursively defined by:



                                        $a_n+1 =|a|a_n$, with $a_1=|a|$.



                                        5) $lim_n rightarrow infty a_n+1 = lim_n rightarrow infty(|a|a_n)$.



                                        With $lim_n rightarrow infty a_n+1 =$



                                        $ lim_n rightarrow infty a_n =:L ge 0$,



                                        we have:



                                        6) $L=|a|L$ with $0 < |a| lt 1 $.



                                        Hence?






                                        share|cite|improve this answer

























                                          up vote
                                          1
                                          down vote










                                          up vote
                                          1
                                          down vote









                                          For fun:



                                          1) $a_n= |a|^n$ is a strictly decreasing sequence for $ 0 <|a| lt 1$.



                                          2) $a_n$ is bounded below by $0$.



                                          3) Hence convergent.



                                          4)Recursively defined by:



                                          $a_n+1 =|a|a_n$, with $a_1=|a|$.



                                          5) $lim_n rightarrow infty a_n+1 = lim_n rightarrow infty(|a|a_n)$.



                                          With $lim_n rightarrow infty a_n+1 =$



                                          $ lim_n rightarrow infty a_n =:L ge 0$,



                                          we have:



                                          6) $L=|a|L$ with $0 < |a| lt 1 $.



                                          Hence?






                                          share|cite|improve this answer















                                          For fun:



                                          1) $a_n= |a|^n$ is a strictly decreasing sequence for $ 0 <|a| lt 1$.



                                          2) $a_n$ is bounded below by $0$.



                                          3) Hence convergent.



                                          4)Recursively defined by:



                                          $a_n+1 =|a|a_n$, with $a_1=|a|$.



                                          5) $lim_n rightarrow infty a_n+1 = lim_n rightarrow infty(|a|a_n)$.



                                          With $lim_n rightarrow infty a_n+1 =$



                                          $ lim_n rightarrow infty a_n =:L ge 0$,



                                          we have:



                                          6) $L=|a|L$ with $0 < |a| lt 1 $.



                                          Hence?







                                          share|cite|improve this answer















                                          share|cite|improve this answer



                                          share|cite|improve this answer








                                          edited Aug 2 at 11:34


























                                          answered Aug 2 at 11:29









                                          Peter Szilas

                                          7,8402617




                                          7,8402617




















                                              up vote
                                              0
                                              down vote













                                              Note that $$ |a^n+1| = |a||a^n|$$



                                              The sequence $a^n$ is decreasing and bounded below so it converges to $L$



                                              Taking limit of $$ |a^n+1| = |a||a^n|$$ we get $|a|L=L$ which implies $L=0$






                                              share|cite|improve this answer

























                                                up vote
                                                0
                                                down vote













                                                Note that $$ |a^n+1| = |a||a^n|$$



                                                The sequence $a^n$ is decreasing and bounded below so it converges to $L$



                                                Taking limit of $$ |a^n+1| = |a||a^n|$$ we get $|a|L=L$ which implies $L=0$






                                                share|cite|improve this answer























                                                  up vote
                                                  0
                                                  down vote










                                                  up vote
                                                  0
                                                  down vote









                                                  Note that $$ |a^n+1| = |a||a^n|$$



                                                  The sequence $a^n$ is decreasing and bounded below so it converges to $L$



                                                  Taking limit of $$ |a^n+1| = |a||a^n|$$ we get $|a|L=L$ which implies $L=0$






                                                  share|cite|improve this answer













                                                  Note that $$ |a^n+1| = |a||a^n|$$



                                                  The sequence $a^n$ is decreasing and bounded below so it converges to $L$



                                                  Taking limit of $$ |a^n+1| = |a||a^n|$$ we get $|a|L=L$ which implies $L=0$







                                                  share|cite|improve this answer













                                                  share|cite|improve this answer



                                                  share|cite|improve this answer











                                                  answered Aug 2 at 9:57









                                                  Mohammad Riazi-Kermani

                                                  27.1k41851




                                                  27.1k41851




















                                                      up vote
                                                      0
                                                      down vote













                                                      Why not just (assuming as mfl did that $0<a<1$) use definitions:
                                                      $a^n$ is decreasing and bounded below, hence convergent, as in mfl's post, and the limit $L = inf a^n$ is pretty clearly nonnegative. Assume by way of contradiction that $L>0$. Then there exists $N>0$ such that $nge N$ implies $a^n < L(1/a)$. Then $a^N+1 < L$, contradiction.






                                                      share|cite|improve this answer

























                                                        up vote
                                                        0
                                                        down vote













                                                        Why not just (assuming as mfl did that $0<a<1$) use definitions:
                                                        $a^n$ is decreasing and bounded below, hence convergent, as in mfl's post, and the limit $L = inf a^n$ is pretty clearly nonnegative. Assume by way of contradiction that $L>0$. Then there exists $N>0$ such that $nge N$ implies $a^n < L(1/a)$. Then $a^N+1 < L$, contradiction.






                                                        share|cite|improve this answer























                                                          up vote
                                                          0
                                                          down vote










                                                          up vote
                                                          0
                                                          down vote









                                                          Why not just (assuming as mfl did that $0<a<1$) use definitions:
                                                          $a^n$ is decreasing and bounded below, hence convergent, as in mfl's post, and the limit $L = inf a^n$ is pretty clearly nonnegative. Assume by way of contradiction that $L>0$. Then there exists $N>0$ such that $nge N$ implies $a^n < L(1/a)$. Then $a^N+1 < L$, contradiction.






                                                          share|cite|improve this answer













                                                          Why not just (assuming as mfl did that $0<a<1$) use definitions:
                                                          $a^n$ is decreasing and bounded below, hence convergent, as in mfl's post, and the limit $L = inf a^n$ is pretty clearly nonnegative. Assume by way of contradiction that $L>0$. Then there exists $N>0$ such that $nge N$ implies $a^n < L(1/a)$. Then $a^N+1 < L$, contradiction.







                                                          share|cite|improve this answer













                                                          share|cite|improve this answer



                                                          share|cite|improve this answer











                                                          answered Aug 2 at 10:01









                                                          John Brevik

                                                          1,22748




                                                          1,22748




















                                                              up vote
                                                              0
                                                              down vote













                                                              By ratio test



                                                              $$frac^n+1=|a|<1 implies |a|^n to 0$$



                                                              then since



                                                              $$-|a|^nle a^nle |a|^n$$



                                                              by squeeze theorem we conclude that



                                                              $$a^n to 0$$






                                                              share|cite|improve this answer

























                                                                up vote
                                                                0
                                                                down vote













                                                                By ratio test



                                                                $$frac^n+1=|a|<1 implies |a|^n to 0$$



                                                                then since



                                                                $$-|a|^nle a^nle |a|^n$$



                                                                by squeeze theorem we conclude that



                                                                $$a^n to 0$$






                                                                share|cite|improve this answer























                                                                  up vote
                                                                  0
                                                                  down vote










                                                                  up vote
                                                                  0
                                                                  down vote









                                                                  By ratio test



                                                                  $$frac^n+1=|a|<1 implies |a|^n to 0$$



                                                                  then since



                                                                  $$-|a|^nle a^nle |a|^n$$



                                                                  by squeeze theorem we conclude that



                                                                  $$a^n to 0$$






                                                                  share|cite|improve this answer













                                                                  By ratio test



                                                                  $$frac^n+1=|a|<1 implies |a|^n to 0$$



                                                                  then since



                                                                  $$-|a|^nle a^nle |a|^n$$



                                                                  by squeeze theorem we conclude that



                                                                  $$a^n to 0$$







                                                                  share|cite|improve this answer













                                                                  share|cite|improve this answer



                                                                  share|cite|improve this answer











                                                                  answered Aug 2 at 10:13









                                                                  gimusi

                                                                  63.8k73480




                                                                  63.8k73480






















                                                                       

                                                                      draft saved


                                                                      draft discarded


























                                                                       


                                                                      draft saved


                                                                      draft discarded














                                                                      StackExchange.ready(
                                                                      function ()
                                                                      StackExchange.openid.initPostLogin('.new-post-login', 'https%3a%2f%2fmath.stackexchange.com%2fquestions%2f2869873%2fproving-limit-of-an-to0-for-a1-without-use-of-logarithms%23new-answer', 'question_page');

                                                                      );

                                                                      Post as a guest













































































                                                                      Comments

                                                                      Popular posts from this blog

                                                                      Color the edges and diagonals of a regular polygon

                                                                      Relationship between determinant of matrix and determinant of adjoint?

                                                                      What is the equation of a 3D cone with generalised tilt?